1

Proof of a cyclic inequality.

Let $a_i$ , $i=1..n$, $n \geq 3$ be real numbers with $a_i > 0 $ and $\prod_{i=1}^n a_i= 1$. Denote $S = \sum_{i=1}^n a_i$ (unrestricted).

Prove (or disprove) the following cyclic (i.e. $a_{n+1} = a_{1}$) inequality at least for $n=4$, preferably for general $n \geq 6$:

$$\sum_{i=1; cyc}^n \frac{a_i \; \{S - (a_i + a_{i+1})\}}{a_i + a_{i+1}} \geq \frac{(n-2)\, n }{2}$$

Remarks:

  1. Obviously, if all $a_i$ are equal, $a_i = a$, we have $a = 1$ due to the product condition and hence one gets equality since $$LHS = \sum_{i=1; cyc}^n \frac{n -2}{2} = \frac{(n -2)n}{2}$$

  2. The case for $n=3$ is given by $$ \frac{a\; c}{a +b} + \frac{b\; a}{b +c} + \frac{c\; b}{c +a} \geq \frac{3}{2}$$ or equivalently, due to the product condition, $$ \frac{1}{b(a +b)} + \frac{1}{c(b +c)} + \frac{1}{a(c +a)} \geq \frac{3}{2}$$

which is known (2008 International Zhautykov Olympiad), for some proofs see here: http://artofproblemsolving.com/community/c6h183916p1010959

  1. In the meantime (29. April 2016), also the case $n=5$ was proven (take into account the comment below by Macavity Mar 25 at 6:36) by the author, see here: Inequality with five variables
Andreas
  • 16,627
  • This is equivalent to $\displaystyle \sum_{cyc} \frac{a_1}{a_1+a_2} \ge 1+\frac{n(n-2)}{2S}$ which is tighter than conjectured here http://math.stackexchange.com/questions/1017110 – Macavity Mar 25 '16 at 06:36
  • True. The RHS (here) is the same as in your link for $S=n$ and the problem (here) is tighter than in the link for $S>n$. With respect to the link, it proves the case $n=3$ (which was claimed there to be proven easily but was never demonstrated) and it may give a handle on the linked problem since the case $n=3$ (here) may be generalized. – Andreas Mar 25 '16 at 10:59

1 Answers1

1

Here is a proof for general $n$.

As pointed out in the comment by Macavity on Mar 25 at 6:36, the claim is equivalent to:

$$\sum_{cyc}\frac{a_i}{a_i+a_{i+1}}\geq 1+\frac{n(n-2)}{2S}$$

We can remove the requirement $\prod_{i=1}^n a_i=1$ by homogenizing this to (simultaneously defining $L$ and $R$): $$L = \sum_{cyc}\frac{a_i}{a_i+a_{i+1}}\geq 1+\frac{n(n-2)\sqrt[n]{a_1a_2...a_n}}{2S} = R$$

The discussion following here is a generalization of the discussion for $n=5$ at Inequality with five variables

We write $2 L \geq 2 R$ or $L \geq 2 R- L$ and add on both sides a term $$\sum_{cyc}\frac{a_{i+1}}{a_i+a_{i+1}}$$ which leaves us to show

$$n = \sum_{cyc}\frac{a_i + a_{i+1}}{a_i+a_{i+1}}\geq 2+\frac{n(n-2)\sqrt[n]{a_1a_2...a_n}}{S} + \sum_{cyc}\frac{-a_i + a_{i+1}}{a_i+a_{i+1}}$$

or, in reformulation of our claim,

$$ \sum_{cyc}\frac{-a_i + a_{i+1}}{a_i+a_{i+1}} \leq (n - 2) (1- \frac{n \sqrt[n]{a_1a_2...a_n}}{S} )$$

We will need the following Lemma (required below), which is the above L-R-inequality for 3 variables:

$$ \frac{b-a}{b+a} + \frac{c-b}{c+b} + \frac{a-c}{a+c} \leq (1- \frac{3 \sqrt[3]{a\, b \, c}}{a + b+ c} )$$

This Lemma is, from the above discussion, just a re-formulation of the claim in $L$ and $R$ above, for 3 variables, i.e.

$$ \frac{a}{b+a} + \frac{b}{c+b} + \frac{c}{a+c} \geq 1+\frac{3\sqrt[3]{a \, b \ c}}{2(a+b+c)}$$

By homogeneity, we can demand $abc=1$ and under that restriction, we need to show $$ \frac{a}{b+a} + \frac{b}{c+b} + \frac{c}{a+c} \geq 1+\frac{3}{2(a+b+c)}$$

This reformulates into $$ \frac{a\; c}{a +b} + \frac{b\; a}{b +c} + \frac{c\; b}{c +a} \geq \frac{3}{2}$$ which is known (see remark 2 in the problem description). Hence the Lemma holds.

For general $n$, we rewrite the LHS of our above reformulation by adding and subtracting terms $\frac{a_{k}-a_1}{a_{k}+a_1}$, for $k=3 \ldots (n-1)$. Here, the first terms are given by $a,b,c,d,e$ for better readability.

$$ \sum_{cyc}\frac{-a_i + a_{i+1}}{a_i+a_{i+1}} = \\ (\frac{b-a}{b+a} + \frac{c-b}{c+b} + \frac{a-c}{a+c}) + (\frac{c-a}{c+a}+\frac{d-c}{d+c} + \frac{a-d}{a+d}) + (\frac{d-a}{d+a}+ \frac{e-d}{e+d} + \frac{a-e}{a+e}) \\ + \cdots + (\frac{a_{n-1}-a_1}{a_{n-1}+a_1}+ \frac{a_{n}-a_{n-1}}{a_{n}+a_{n-1}} + \frac{a_1-a_n}{a_1+a_n}) $$ There are in total $n-2$ of these terms with three summands each. This also holds for any of the $n$ cyclic shifts in $(a_1 \cdots a_n)$, denoted by $cyc(n)$, so we can write

$$ n \sum_{cyc}\frac{-a_i + a_{i+1}}{a_i+a_{i+1}} = \\ \sum_{cyc (n)} (\frac{b-a}{b+a} + \frac{c-b}{c+b} + \frac{a-c}{a+c}) + \sum_{cyc (n)}(\frac{c-a}{c+a}+\frac{d-c}{d+c} + \frac{a-d}{a+d}) + \sum_{cyc (n)} (\frac{d-a}{d+a}+ \frac{e-d}{e+d} + \frac{a-e}{a+e}) + \cdots + \sum_{cyc (n)} (\frac{a_{n-1}-a_1}{a_{n-1}+a_1}+ \frac{a_{n}-a_{n-1}}{a_{n}+a_{n-1}} + \frac{a_1-a_n}{a_1+a_n}) $$

Since all triples of terms are in 3 variables only, we can use our Lemma. Then it suffices to show

$$ \sum_{cyc (n)} (1- \frac{3 \sqrt[3]{a\, b \, c}}{a + b+ c} ) + \sum_{cyc (n)}(1- \frac{3 \sqrt[3]{a\, c \, d}}{a + c+ d} ) + \sum_{cyc (n)}(1- \frac{3 \sqrt[3]{a\, d \, e}}{a + d+ e} ) + \cdots + \sum_{cyc (n)}(1- \frac{3 \sqrt[3]{a_1\, a_{n-1} \, a_n}}{a_1 + a_{n-1} + a_n} )\\ \leq n (n-2) (1- \frac{n \sqrt[n]{a_1a_2...a_n}}{S} ) $$ which is

$$ \sum_{cyc (n)} (\frac{3 \sqrt[3]{a\, b \, c}}{a + b+ c} ) + \sum_{cyc (n)}(\frac{3 \sqrt[3]{a\, c \, d}}{a + c+ d} ) + \sum_{cyc (n)}(\frac{3 \sqrt[3]{a\, d \, e}}{a + d+ e} ) + \cdots + \sum_{cyc (n)}(\frac{3 \sqrt[3]{a_1\, a_{n-1} \, a_n}}{a_1 + a_{n-1} + a_n} )\\ \geq n^2 \, (n-2) (\frac{\sqrt[n]{a_1a_2...a_n}}{S} ) $$ Using Cauchy-Schwarz leaves us with showing $$ \frac {(\sum_{cyc (n)} \sqrt[6]{a\, b \, c})^2}{\sum_{cyc (n)}(a + b+ c)} + \frac {(\sum_{cyc (n)} \sqrt[6]{a\, c \, d})^2}{\sum_{cyc (n)}(a + c+ d)} + \frac {(\sum_{cyc (n)} \sqrt[6]{a\, d \, e})^2}{\sum_{cyc (n)}(a + d+ e)} + \cdots + \frac {(\sum_{cyc (n)} \sqrt[6]{a_1\, a_{n-1} \, a_n})^2}{\sum_{cyc (n)}(a_1 + a_{n-1} + a_n)} \geq \frac{1}{3} n^2 \,(n-2) \frac{\sqrt[n]{a_1a_2...a_n}}{S} $$

The denominators all equal $3S$, so this becomes $$ {(\sum_{cyc (n)} \sqrt[6]{a\, b \, c})^2} +{(\sum_{cyc (n)} \sqrt[6]{a\, c \, d})^2} +{(\sum_{cyc (n)} \sqrt[6]{a\, d \, e})^2} + \cdots + {(\sum_{cyc (n)} \sqrt[6]{a_1\, a_{n-1} \, a_n})^2}\\ \geq n^2 \,(n-2) \sqrt[n]{a_1a_2...a_n} $$ Using AM-GM gives for the first term

$$ (\sum_{cyc (n)} \sqrt[6]{a\, b \, c})^2 \geq ( n (\prod_{cyc (n)} \sqrt[6]{a\, b \, c} )^{1/n})^2 = n^2 (\prod_{cyc (n)} ({a\, b \, c} ) )^{1/(3 n)} = n^2 \sqrt[n]{a_1a_2...a_n} $$

By the same procedure, the other term on the LHS are likewise greater or equal than $n^2 \sqrt[n]{a_1a_2...a_n}$. There are $n-2$ of these terms, hence the sum on the LHS after AM-GM equals the RHS, which concludes the proof.

Andreas
  • 16,627